Difference between revisions of "2009 AMC 10A Problems/Problem 1"

(Problem)
(Solution 2)
Line 12: Line 12:
  
 
== Solution 2 ==
 
== Solution 2 ==
We can divide <math>128/12</math> and round up because there are a whole number of cans.
+
We want to find <math>\left\lceiling\frac{128}{12}\right\rceiling because there are a whole number of cans.
  
<math>128/12 = 10R8\longrightarrow 11\longrightarrow \fbox{E}.</math>
+
</math>\frac{128}{12} = 10R8\longrightarrow 11\longrightarrow \fbox{E}.$
  
 
{{AMC10 box|year=2009|ab=A|before=First Question|num-a=2}}
 
{{AMC10 box|year=2009|ab=A|before=First Question|num-a=2}}
 
{{MAA Notice}}
 
{{MAA Notice}}

Revision as of 07:41, 6 May 2018

Problem

One can, can hold $12$ ounces of soda, what is the minimum number of cans needed to provide a gallon ($128$ ounces) of soda?

$\textbf{(A)}\ 7\qquad \textbf{(B)}\ 8\qquad \textbf{(C)}\ 9\qquad \textbf{(D)}\ 10\qquad \textbf{(E)}\ 11$

Solution 1

$10$ cans would hold $120$ ounces, but $128>120$, so $11$ cans are required. Thus, the answer is $\mathrm{(E)}$.

Solution 2

We want to find $\left\lceiling\frac{128}{12}\right\rceiling because there are a whole number of cans.$ (Error compiling LaTeX. Unknown error_msg)\frac{128}{12} = 10R8\longrightarrow 11\longrightarrow \fbox{E}.$

2009 AMC 10A (ProblemsAnswer KeyResources)
Preceded by
First Question
Followed by
Problem 2
1 2 3 4 5 6 7 8 9 10 11 12 13 14 15 16 17 18 19 20 21 22 23 24 25
All AMC 10 Problems and Solutions

The problems on this page are copyrighted by the Mathematical Association of America's American Mathematics Competitions. AMC logo.png